Đến nội dung

nguyentrungphuc26041999 nội dung

Có 382 mục bởi nguyentrungphuc26041999 (Tìm giới hạn từ 30-03-2020)



Sắp theo                Sắp xếp  

#683062 $n^{13}+n^{5}+1$

Đã gửi bởi nguyentrungphuc26041999 on 04-06-2017 - 16:49 trong Số học

Phải là $n^{4}$ mới đúng chứ nhỉ

đề nó ghi như thế ông ạ




#683055 $n^{13}+n^{5}+1$

Đã gửi bởi nguyentrungphuc26041999 on 04-06-2017 - 16:25 trong Số học

Tìm số nguyên dương n để $n^{13}+n^{5}+1$ là số nguyên tố




#650854 $\left ( m^{n}-1 \right )\vdots k^{m}...

Đã gửi bởi nguyentrungphuc26041999 on 22-08-2016 - 21:23 trong Số học

tìm tất cả các bộ số $\left ( k,m,n \right )$ sao cho $\left ( m^{n}-1 \right )\vdots k^{m}$ và $\left ( n^{m}-1 \right )\vdots k^{n}$




#537982 $\left\{\begin{matrix} 2x+1=\left ( y...

Đã gửi bởi nguyentrungphuc26041999 on 14-12-2014 - 22:44 trong Phương trình, hệ phương trình và bất phương trình

đây là 1 hệ đối xứng vô cùng quen thuộc bạn chỉ cần đặt y+1=z là giải dc dễ dàng( bằng cách trừ 2 vế của 2 pt)

viết ra đi




#537972 $\left\{\begin{matrix} 2x+1=\left ( y...

Đã gửi bởi nguyentrungphuc26041999 on 14-12-2014 - 22:34 trong Phương trình, hệ phương trình và bất phương trình

giải hệ phương trình

$\left\{\begin{matrix} 2x+1=\left ( y+1 \right )^{3} & \\ x^{3}=3y+2 & \end{matrix}\right.$




#521727 Cho a,b,c là các số thuộc đoạn [1;2].Chứng minh rằng:$(a+b+c)(\frac...

Đã gửi bởi nguyentrungphuc26041999 on 28-08-2014 - 21:23 trong Bất đẳng thức và cực trị

Cho a,b,c là các số thuộc đoạn [1;2].Chứng minh rằng:$(a+b+c)(\frac{1}{a}+\frac{1}{b}+\frac{1}{c})\leq 10$ 

bất đẳng thức tương đương với $\sum \frac{a}{b}+\sum \frac{b}{a}\leq 7$

không mất tính tổng quát giả sử $a\geq b\geq c$

$\Rightarrow \left ( a-b \right )\left ( b-c \right )\geq 0$

$\Rightarrow ab+bc\geq b^{2}+ac$

$\frac{a}{c}+1\geq \frac{b}{c}+\frac{a}{b}$ và $\frac{c}{a}+1\geq \frac{b}{a}+\frac{c}{b}$

cộng 2 vế lại,bây giờ ta cần chứng minh $2\left ( \frac{c}{a}+\frac{a}{c} \right )+2\leq 7$

cái này thì đặt $frac{a}{c}=x$ rồi biến đổi tuơng đương kết hợp với dk là ta có đpcm




#515189 Cho A=2+2$\sqrt{12n^{2}+1}$ là số nguyên (với n thuộc N) thì A...

Đã gửi bởi nguyentrungphuc26041999 on 24-07-2014 - 19:49 trong Số học

Giải như sau:
Để A nguyên thì $\sqrt{12n^2+1}$ nguyên suy ra $12n^2+1$ là số chính phương.
Đặt $12n^2+1=a^2$ như vậy $12n^2=(a-1)(a+1)$
Dễ thấy $(a-1)(a+1)$ chia hết cho 12 suy ra $(a-1);(a+1)$ cùng chẵn nên đặt $a-1=2x,a+1=2y$ <2>
Nhưng lại thấy $gcd(a-1,a+1)=1,2$ mà chúng cùng chắn nên $gcd(a-1,a+1)=2$ do vậy $gcd(x,y)=1$ <1>
Do vậy $3n^2=xy$
Th1: $x$ chia hết cho 3 suy ra theo <1> thì $y$ không chia hết cho 3
Đặt $x=3k$ cũng suy ra $gcd(k,y)=1$ và $n^2=ky$ sở dĩ k,y nguyên tố cùng nhay suy ra mỗi số là chính phương
Do vậy $n^2=p^2.q^2;k=p^2,y=q^2$ như vậy theo <2> thì $a=2q^2-1$
Như vậy $A=2+\sqrt{12n^2+1}=2+2a=4q^2$ là số chính phương $đpcm$
Th2: $y$ chia hết cho 3 suy ra $y=3t$ và xét tương tự như trên.
Vậy ta có bài toán được chứng minh hoàn toàn

không tương tự cho lắm anh ạ




#506777 Sáng tạo Bất đẳng thức _ Phạm Kim Hùng quyển 2

Đã gửi bởi nguyentrungphuc26041999 on 15-06-2014 - 07:48 trong Tài nguyên Olympic toán

Chào mọi người, ắt hẳn diendantoanhoc.net ta không thể không biết đến cuốn Sáng tạo BĐT của anh PKH (hungkhtn). Trước đây đã có bản file tập 1(về cả Tiếng Anh lẫn Tiếng Việt). Bây giờ xin giới thiệu với mọi người file tập 2 của cuốn này, xuất bản bởi Gil vào năm 2009. Cuốn sách tập 2, Secrets In Inequalities, Advance Inequalities hay hơn tập 1 nhiều, vì nó mới là phần chính của 2 tập.

secrets_in_inequalities_-_vol_2_1.jpg

Download

Đã có:
Secrets in Inequalities I, Nhà xuất bản Gil, 2008. Download (English)

Sáng tạo bất đẳng thức, Nhà xuất bản Tri Thức, Hà Nội, 2007. Download. (VIETnamese)

sao không down được,trang có vấn đề à Toàn




#506437 Cho các số thực dương x,y Chứng minh rằng : a, $\frac{a}...

Đã gửi bởi nguyentrungphuc26041999 on 13-06-2014 - 22:03 trong Bất đẳng thức và cực trị

Sao có thể khẳng định $\left\{\begin{matrix} a+b=x & \\ b=y & \end{matrix}\right.$ , đề chỉ cho $x,y$ là các số thực dương liên quan gì đến $a,b$ 

ý mình là có lẽ đề ra cho a,b chứ ko phải a,ở đây mình đặt ẩn phụ mà




#506425 Cho các số thực dương x,y Chứng minh rằng : a, $\frac{a}...

Đã gửi bởi nguyentrungphuc26041999 on 13-06-2014 - 21:51 trong Bất đẳng thức và cực trị

có lạc đề không đó bạn

thay $x,y$ vào là xong là thấy đúng đề




#506415 Cho các số thực dương x,y Chứng minh rằng : a, $\frac{a}...

Đã gửi bởi nguyentrungphuc26041999 on 13-06-2014 - 21:34 trong Bất đẳng thức và cực trị

 

Cho các số thực dương x,y Chứng minh rằng :

a, $\frac{a}{4b^{2}}+\frac{2b}{(a+b)^{2}}\geq \frac{9}{4(a+2b)}$

b, $\frac{2}{a^{2}+ab+b^{2}}+\frac{1}{3b^{2}}\geq \frac{9}{(a+2b)^{2}}$

 

a,

Đặt$\left\{\begin{matrix} a+b=x & \\ b=y & \end{matrix}\right.$

$\Leftrightarrow \frac{x-y}{y^{2}}+\frac{8y}{x^{2}}\geq \frac{9}{x+y}$

$\Leftrightarrow \frac{x^{2}}{y^{2}}+\frac{8y}{x}+\frac{8y^{2}}{x^{2}}\geq 10$

áp dụng bất đẳng thức côsi

$\left ( \frac{x^{2}}{2y^{2}}+\frac{8y^{2}}{x^{2}} \right )+\left (\frac{x^{2}}{2y^{2}}+\frac{4y}{x}+\frac{4y}{x} \right )\geq 10$




#504792 Chứng minh rằng: $ a^2+b^2+c^2\leq 14$

Đã gửi bởi nguyentrungphuc26041999 on 07-06-2014 - 20:37 trong Bất đẳng thức và cực trị

Cho $1\leq a,b,c\leq 3 $ thỏa mãn :$a+b+c=6$

Chứng minh rằng: $ a^2+b^2+c^2\leq 14$

đặt $a=x+1,b=y+1,c=z+1$

$0\leq x,y,z\leq 2,x+y+z=3$

ta có $a^{2}+b^{2}+c^{2}=x^{2}+y^{2}+z^{2}+2\left ( x+y+z \right )+3=x^{2}+y^{2}+z^{2}+9$

ta có $\left ( 2-x \right )\left ( 2-y \right )\left ( 2-z \right )\geq 0$

nhân tung ra là xong




#502014 $\left\{\begin{matrix} \sqrt{x^...

Đã gửi bởi nguyentrungphuc26041999 on 27-05-2014 - 20:30 trong Phương trình - hệ phương trình - bất phương trình

Giải hệ phương trình: 

$\left\{\begin{matrix} \sqrt{x^{2}-y}=z-1 & & \\ \sqrt{y^{2}-z}=x-1& & \\ \sqrt{z^{2}-x}=y-1& & \end{matrix}\right.$

Ta có $x+y+z\geq 3$

Phương trình tương đương $\left\{\begin{matrix} x^{2}-y=z^{2}-2z+1 & \\ y^{2}-z=x^{2}-2x+1 & \\ z^{2}-x=y^{2}-2y+1 & \end{matrix}\right.$

$\Leftrightarrow x+y+z=3$

mà $x\geq 1,y\geq 1,z\geq 1$

$\Rightarrow x=y=z=1$




#501993 $B=\frac{x}{y+z}+\frac{y}{z...

Đã gửi bởi nguyentrungphuc26041999 on 27-05-2014 - 19:44 trong Bất đẳng thức và cực trị

Bài 1:Giả sử $ c = min (a,b,c) $.Ta có:$0 <a-c \leq a$ và $0<b-c \leq b$

Khi đó,ta có: $P \geq (a^2+b^2)[\frac{1}{(a-b)^2}+\frac{1}{a^2}+\frac{1}{b^2}] =\frac{a^2+b^2}{(a-b)^2}+(a^2+b^2)(\frac{1}{a^2}+\frac{1}{b^2})$

Đặt :$\frac{a}{b}+\frac{b}{a}=x,x>2$.Khi đó ta có:

$\frac{a^2+b^2}{(a-b)^2}=\frac{a^2+b^2}{a^2+b^2-2ab}=\frac{\frac{a}{b}+\frac{b}{a}}{\frac{a}{b}+\frac{b}{a}-2}=\frac{x}{x-2}$

$(a^2+b^2)(\frac{1}{a^2}+\frac{1}{b^2}) =\frac{a^2}{b^2}+\frac{b^2}{a^2}+2 =x^2$

Suy ra: $P \geq \frac{x}{x-2}+x^2$.

Xét hàm số: $f(x) =\frac{x}{x-2}+x^2$ với $x>2$

Ta có:$f'(x)=\frac{2(x-1)(x^2-3x+1)}{(x-2)^2}$.Ta có:$f'(x)=0$ $\Leftrightarrow$ $x=\frac{3+\sqrt{5}}{2} >2$

Hàm $f(x)$ nghịch biến trên khoảng $(2;\frac{3+\sqrt{5}}{2})$ và đồng biến trên khoảng $(\frac{3+\sqrt{5}}{2};+\infty)$

Do đó:$MinA=Minf(x)_{x \in (2;+\infty)} =f(\frac{3+\sqrt{5}}{2}) =\frac{11+5\sqrt{5}}{2}$.Dấu $"="$ xẩy ra $\Leftrightarrow$ $c=0$ và $\frac{a}{b}+\frac{b}{a}=\frac{3+\sqrt{5}}{2}$ cùng các hoán vị

Chẳng hạn,khi cho $b=1$ thì dấu $"="$ xẩy ra tại $(a;b;c)=(\frac{3+\sqrt{5}+\sqrt{6\sqrt{5}-2}}{4};1;0)$

Bài 2:Theo giả thết ta có:$x+y-z \geq 1$ $\Leftrightarrow$ $x+y \geq 1+z$

Ta có: $B =\frac{x}{y+z}+\frac{y}{z+x}+\frac{z}{xy+z^2} =\frac{x^2}{x(y+z)}+\frac{y^2}{y(z+x)}+\frac{z}{xy+z^2}$

Áp dụng Cauchy-Schwars ta có: $B \geq \frac{(x+y)^2}{2xy+z(x+y)}+\frac{z}{xy+z^2} (1)$

Đặt :$t=x+y$,theo giả thiết ta có:$1+z \leq t \leq 2$ và $xy \leq \frac {t^2}{4} (2)$

Theo $(1)$ và $(2)$ ta suy ra được:$B \geq \frac{2t^2}{t^2+2zt}+\frac{4z}{t^2+z^2}=f(t)$.Xét hàm $f(t)$ trên $[1+z;2]$ ta có

$f'(t) = 4zt [\frac{t}{ (t^2+2zt)^2}-\frac{2}{(t^2+4z^2)}]$,mặt khác do $t \geq z+1$ và $z \leq 1$ nên $2zt \geq 4z^2$ suy ra $\frac{t}{(t^2+2zt )^2}\leq \frac{2}{t^2+4z^2}$ $\Rightarrow$ hàm $f(t)$ nghịch biến với mọi $t \in [z+1;2]$ $\Rightarrow$ $f(t) \geq f(2)=\frac{2}{1+z}+\frac{z}{z^2+1}=g(z)$

Khảo sát hàm $g(z)$  trên $(0;1]$ ta có:$g'(z)=-\frac{2}{(1+z)^2} +\frac{1-z^2}{(z^2+1)^2} \leq 0$ với mọi $z \in (0;1]$.Suy ra,hàm $g(z)$ nghich biến trên $(0;1]$

Suy ra,$g(z) \geq g(1) =\frac{3}{2}$

Vậy,$MinB=\frac{3}{2}$,dấu $"="$ xẩy ra $\Leftrightarrow$ $x=y=z=1$

$x>2$ mà $x=1$ à,sai rồi kìa,còn bài này mình đã học từ năm ngoái,đúng chứ không sai được




#501773 $\frac{a}{b}+\frac{b}{c...

Đã gửi bởi nguyentrungphuc26041999 on 26-05-2014 - 17:25 trong Bất đẳng thức và cực trị

sao lại có $+1$ nhỉ

fix rồi




#501770 $\frac{a}{b}+\frac{b}{c...

Đã gửi bởi nguyentrungphuc26041999 on 26-05-2014 - 17:13 trong Bất đẳng thức và cực trị

Bất đẳng thức tương đương

$\frac{a}{b}+\frac{b}{c}+\frac{c}{a}+1\geq \frac{a+b}{b+c}+\frac{b+c}{a+b}+2$

$\Rightarrow \frac{a^{2}}{ab}+\frac{b^{2}}{bc}+\frac{c^{2}}{ca}+\frac{b^{2}}{b^{2}}\geq \frac{\left ( a+b+b+c \right )^{2}}{\left ( b+a \right )\left ( b+c \right )}$

hiển nhiên đúng theo Côsi-Schwart




#501766 $B=\frac{x}{y+z}+\frac{y}{z...

Đã gửi bởi nguyentrungphuc26041999 on 26-05-2014 - 17:07 trong Bất đẳng thức và cực trị

Bài 1:Cho $a,b,c$ là các số thực phân biệt.Tìm giá trị nhỏ nhất của biểu thức $A=(a^2+b^2+c^2)[\frac{1}{(a-b)^2}+\frac{1}{(b-c)^2}+\frac{1}{(c-a)^2}]$

Bài 2:Cho $x,y,z \in (0;1]$ thoả mãn $x+y-z \geq 1$.Tìm giá trị  nhỏ nhất của $B=\frac{x}{y+z}+\frac{y}{z+x}+\frac{z}{xy+z^2}$

Bài này dùng nhiều bổ đề kinh

Thế này

Đặt $\frac{a}{b-c}=x,\frac{b}{c-a}=y,\frac{c}{a-b}=z$

ta có $\Rightarrow xy+yz+zx=-1$

mà $x^{2}+y^{2}+z^{2}\geq -2\left ( xy+yz+zx \right )$

$\Rightarrow \sum \frac{a}{\left ( b-c \right )^{2}}\geq 2$

$\Rightarrow \sum \left ( \frac{a^{2}}{b-c}+1 \right )\geq 5$

$\Rightarrow \left ( a^{2}+b^{2}+c^{2} \right )\left ( \sum \frac{1}{\left ( b-c \right )^{2}} \right )\geq 5+2\left ( \sum \frac{bc}{\left ( b-c \right )^{2}} \right )$

ta lại có $\sum \frac{a+b}{a-b}.\frac{b+c}{b-c}=-1$

tiếp tục áp dụng $x^{2}+y^{2}+z^{2}\geq -2\left ( xy+yz+zx \right )$

$\Rightarrow \sum \left ( \frac{a+b}{a-b} \right )^{2}\geq 2$

thêm bớt thôi $\Rightarrow \sum \left ( \frac{a+b}{a-b} \right )^{2}-3\geq -1$

$\Rightarrow \sum \frac{ab}{\left ( a-b \right )^{2}}\geq \frac{-1}{4}$

$\Rightarrow 5+2\sum \frac{bc}{\left ( b-c \right )^{2}}\geq \frac{9}{2}$

Vậy $MIN A=\frac{9}{2}$

 dấu bằng xảy ra khi $x+y+z=0$ cái này bạn giải ra nhé




#499767 $\frac{3a}{3+a}+\frac{4b}{4...

Đã gửi bởi nguyentrungphuc26041999 on 18-05-2014 - 09:58 trong Bất đẳng thức và cực trị

Giải:

Không mất tính tổng quát, chuẩn hóa $a+b+c=12$.

Khi đó, BĐT cần chứng minh tương đương với:

$\frac{3a}{3+a}+\frac{4b}{4+b}+\frac{5c}{5+c}\leq 6$

Ta chứng minh BĐT phụ sau:

$\frac{3a}{3+a}\leq \frac{3+a}{4}\Leftrightarrow -(a-3)^2\leq 0$ (luôn đúng)

$\frac{4b}{4+b}\leq \frac{4+b}{4}\Leftrightarrow -(b-4)^2\leq 0$ (luôn đúng)

$\frac{5c}{5+c}\leq \frac{5+c}{4}\Leftrightarrow -(c-5)^2\leq 0$ (luôn đúng)

Cộng vế các BĐT trên lại ta được đpcm

Dấu "=" khi và chỉ khi $a=3;b=4;c=5$.

P/s: Không biết có đúng không, buồn ngủ quá...!

hình như là không được phép chuẩn hoá vì nó chưa thuần nhất




#499766 TOPIC các bài đất đẳng thức THCS

Đã gửi bởi nguyentrungphuc26041999 on 18-05-2014 - 09:57 trong Bất đẳng thức và cực trị

Bài tiếp :

5) Cho $a,b,c$ là độ dài cạnh của 1 tam giác.

Chứng minh rằng : $\frac{a}{b+c-a}+\frac{b}{a+c-b}+\frac{c}{a+b-c}\geq 3$ (Giải bằng 2 cách)  ~O)

$\sum \frac{a}{b+c-a}\geq 3\sqrt[3]{\frac{abc}{\prod \left ( a+b-c \right )}}\geq 3$

hoặc dùng Svacs




#498452 CMR: $\sum \sqrt{x(y+z)}\geqslant 2\sqrt...

Đã gửi bởi nguyentrungphuc26041999 on 11-05-2014 - 21:05 trong Bất đẳng thức và cực trị

Cho các số thực dương $x,y,z$. CMR: $\sum \sqrt{x(y+z)}\geqslant 2\sqrt{\frac{(x+y)(y+z)(z+x)}{x+y+z}}$

Hình như bài này dấu ngươc lại chứ ạ 

thay $x=y=z$ vào cũng không được nên em sửa đề

Tình hình là thế này 

$2\sqrt{\frac{\left ( x+y \right )\left ( y+z \right )\left ( z+x \right )}{x+y+z}}=2\sqrt{\frac{\left ( x+y+z \right )\left ( xy+yz+zx \right )-xyz}{x+y+z}}=2\sqrt{xy+yz+xz-\frac{xyz}{x+y+z}}$

$=2\sqrt{xy+yz+zx-\frac{1}{\frac{1}{xy}+\frac{1}{xz}+\frac{1}{yz}}}$

đặt $\frac{1}{xy}=a,\frac{1}{yz}=b,\frac{1}{zx}=c$

ta chứng minh 

$\frac{4}{3\sqrt{3}}\sum \sqrt{\frac{1}{a}+\frac{1}{b}}\leq 2\sqrt{\frac{1}{a}+\frac{1}{b}+\frac{1}{c}-\frac{1}{a+b+c}}$

cái này nhân bung ra là xong




#498449 Tìm Min $P= 3(x^2+y^2+z^2)-2xyz$

Đã gửi bởi nguyentrungphuc26041999 on 11-05-2014 - 20:40 trong Bất đẳng thức và cực trị

Cho x,y,z là các số thực dương thoả mãn: x+y+z=3

Tìm GTNN của biểu thức $P= 3(x^2+y^2+z^2)-2xyz$

$P\geq \left ( x+y+z \right )^{2}-2\frac{\left ( x+y+z \right )^{3}}{27}=...$




#498359 Topic tổng hợp các bài toán về phương trình nghiệm nguyên.

Đã gửi bởi nguyentrungphuc26041999 on 11-05-2014 - 10:24 trong Số học

220/ 

a. $\sqrt{x}+\sqrt{x+3}=y$

b.Tìm nghiệm nguyên của hệ phương trình

$\left\{\begin{matrix} 2x+3y=8 & & \\ 5y+3z=1 & & \end{matrix}\right.$

c.$.x+y+z=xyz$

d.$\sqrt{x}+\sqrt{y}=\sqrt{1998}$

e.$3x^2+5y^2=12$

f.$3(x^2+y^2+xy)=x+8y$

g.$x(x+1)(x+2)(x+3)=y^2$

h.$\frac{1}{x^2}+\frac{1}{y^2}+\frac{1}{z^2}+\frac{1}{t^2}=1$

c,giả sử $x\geq y\geq z$

với $x=y=z=0$ đúng

ta có $1=\frac{1}{xy}+\frac{1}{yz}+\frac{1}{xz}\leq \frac{3}{z^{2}}$

$\Rightarrow z=1$

$\Leftrightarrow \left ( x-1 \right )\left ( y-1 \right )=2$




#498355 Topic tổng hợp các bài toán về phương trình nghiệm nguyên.

Đã gửi bởi nguyentrungphuc26041999 on 11-05-2014 - 10:14 trong Số học

220/ 

a. $\sqrt{x}+\sqrt{x+3}=y$

b.Tìm nghiệm nguyên của hệ phương trình

$\left\{\begin{matrix} 2x+3y=8 & & \\ 5y+3z=1 & & \end{matrix}\right.$

c.$.x+y+z=xyz$

d.$\sqrt{x}+\sqrt{y}=\sqrt{1998}$

e.$3x^2+5y^2=12$

f.$3(x^2+y^2+xy)=x+8y$

g.$x(x+1)(x+2)(x+3)=y^2$

h.$\frac{1}{x^2}+\frac{1}{y^2}+\frac{1}{z^2}+\frac{1}{t^2}=1$

a,$\Leftrightarrow 2x+3+2\sqrt{x\left ( x+3 \right )}=y^{2}$

nếu $x=0$  không thoả mãn 

nếu $x=3$ không thoả mãn

nếu $x\left ( x+3 \right )=k^{2}$

$\Leftrightarrow \left ( 2x+3-k \right )\left ( 2x+3+k \right )=9$




#498353 Topic tổng hợp các bài toán về phương trình nghiệm nguyên.

Đã gửi bởi nguyentrungphuc26041999 on 11-05-2014 - 10:07 trong Số học

220/ 

a. $\sqrt{x}+\sqrt{x+3}=y$

b.Tìm nghiệm nguyên của hệ phương trình

$\left\{\begin{matrix} 2x+3y=8 & & \\ 5y+3z=1 & & \end{matrix}\right.$

c.$.x+y+z=xyz$

d.$\sqrt{x}+\sqrt{y}=\sqrt{1998}$

e.$3x^2+5y^2=12$

f.$3(x^2+y^2+xy)=x+8y$

g.$x(x+1)(x+2)(x+3)=y^2$

h.$\frac{1}{x^2}+\frac{1}{y^2}+\frac{1}{z^2}+\frac{1}{t^2}=1$

g,$\Leftrightarrow \left ( x^{2}+3x+2 \right )\left ( x^{2}+3x \right )=y^{2}$

$\left ( x^{2}+3x+1-y \right )\left ( x^{2}+3x+1+y \right )=1$




#498222 Trận 9 - Bất đẳng thức

Đã gửi bởi nguyentrungphuc26041999 on 10-05-2014 - 15:03 trong Thi giải toán Marathon cấp THCS 2014

Cho $x, y, z$ là các số dương thỏa mãn điều kiện $xyz = 1$. Tìm giá trị nhỏ nhất của biểu thức:  

$$E=\frac{1}{x^3(y+z)}+\frac{1}{y^3(z+x)}+\frac{1}{z^3(x+y)}.$$

Toán thủ ra đề: angleofdarkness

Áp dụng bất đẳng thức Bunhiacopxki dạng phân thức,ta có

$E=\sum \frac{\frac{1}{x^{2}}}{xy+xz}\geq \frac{\left ( \frac{1}{x} +\frac{1}{y}+\frac{1}{z}\right )^{2}}{2\left ( xy+yz+zx \right )}=\frac{xy+yz+zx}{2}$

áp dụng bất đẳng thức Cô-si 

$\frac{xy+yz+xz}{2}\geq \frac{3\sqrt[3]{x^{2}y^{2}z^{2}}}{2}=\frac{3}{2}$

Vậy giá trị nhỏ nhất của $E=\frac{3}{2}$ đạt được khi $a=b=c=1$

 

$d = 10$

$S = 41$